7k -2(k+4) What is this answer?

Answers

Answer 1

5−8

llllllllllllllllllllllllllllllllllllllllllllllllllllllllllllllllllllllllllllllllllllll


Related Questions

Evaluate the algebraic expression if w= -0.5, x = 2, y= -3, and z = 2.5.

2w^2 - 3x + y - 8z

Answers

Answer:

-28.50

Step-by-step explanation:

Given the algebraic expression, 2w² - 3x + y - 8z, where w = -0.5, x = 2, y= -3, and z = 2.5:

Substitute the given values for the variables to evaluate the algebraic expression.

2w² - 3x + y - 8z

= 2(-0.5)² - 3(2) + (-3) - 8(2.5)

= 2(0.25) - 6 - 3 - 20

= 0.50 - 6 - 3 - 20

= -28.50

Therefore, 2w² - 3x + y - 8z = -28.50.

Answer:

-28.5

Step-by-step explanation:

Substitute the values for their given variables and evaluate.

[tex]2w^2-3x+y-8z\\\\2(-0.5)^2-3(2)+(-3)-8(2.5)\\\\2(0.25)-6-3-20\\\\0.5-6-3-20\\\\-28.5[/tex]

find the integral.......

Answers

Answer:

1/5(2ln|x|+3*5^1/3)+c

Step-by-step explanation:

Compare the expressions 7+3^2-2⋅5 and (3/4+1/8)÷1/8 using , =, or . Show your work. Answer:

Answers

[tex]A=7+3^2 -2 \cdot 5 = 7 + 9 - 10 = 9-3=6\\\\\\B = \left(\dfrac 34 + \dfrac 18\right) \div \dfrac 18 = \dfrac 78 \times 8 =7\\\\\\B>A[/tex]

The expression relation is 7+3²-2⋅5 > (3/4+1/8)÷1/8.

What is Inequality?

Mathematical expressions with inequalities are those in which the two sides are not equal. Contrary to equations, we compare two values in inequality. Less than (or less than or equal to), greater than (or greater than or equal to), or not equal to signs are used in place of the equal sign.

Given:

First, 7+3²-2⋅5

Solving the above expression

= 7 + 9 -2.5

= 16 - 2.5

= 13.5

Second,

(3/4+1/8)÷1/8

= (6/8 + 1/8) ÷ 1/8

= 7/8 ÷ 1/8

= 7/8 x 8/1

= 7

Hence, 7+3²-2⋅5 > (3/4+1/8)÷1/8.

Learn more about Inequality here:

https://brainly.com/question/30231190

#SPJ2

What is the value of w?

Answers

Answer:

w= -2. Solve for w: (w-5) (w+2) = w^2-4. Expand out terms of the left hand side: w^2-3 w-10 = w^2-4. Subtract w^2-4 from both sides

Step-by-step explanation:

The graph of a linear function has a negative slope. Describe the end behavior of this function.
As the value of x increases, the value of y [ A. decreases , B. increases ].
As the value of x decreases, the value of y [ A. decreases , B. increases ].

Answers

Answer:

1. A

2. B

Step-by-step explanation:

It would look like y = -m(x) + b.

For example, m is 1 and b is 0. (You can try any random numbers, but I tried to find the simplest one so it's easy for us to see.) The equation will be y= -x. (you can substitute random numbers in here, too)

If x = 3, y = -3 ; x = 5, y = -5  (x increases, y decreases)

If x = -1, y = 1 ; x = -5, y = 5 (x decreases, y increases)

As you can see from the example,

If the value of x increases, the value of y will decrease and vice versa

For the negative slope equation y = -x as the value of x increases y decreases.

What is a slope?

Slope or the gradient is the number or the ratio which determines the direction or the steepness of the line.

Given that the graph of a linear function has a negative slope. Suppose the equation with a negative slope is y = -x.

It would look like y = -m(x) + b.

As an illustration, m is 1 and b is 0. (You can try any random number; however, I looked for the most straightforward one so that we could easily observe it.) It will be written as y=-x. (You can also omit this and use random numbers)

If x = 3, y = -3 ; x = 5, y = -5  (x increases, y decreases)

If x = -1, y = 1 ; x = -5, y = 5 (x decreases, y increases)

Therefore, for the negative slope equation y = -x as the value of x increases y decreases.

To know more about slopes follow

https://brainly.com/question/3493733

#SPJ2


The cost of 2 bunches of
roses and 5 bunches of
carnations is £45.
The cost of 3 bunches of
roses and 4 bunches of
carnations is £50.
Find the cost of one bunch of each

Answers

Answer:

vtdOveractivity of the pituitary gland is called hyperpituitarism. Several disorders related to an overactive pituitary gland can occur.

...

Symptoms may include:

Nervousness.Rapid or irregular heartbeat.Weight loss.Fatigue.Muscular weakness.

you have to estimate​

Answers

The answer is 75388.
802x94
=75388

Answer:

75,388

Step-by-step explanation:

Long Multiplication form

[tex]\mathrm{Mutliply\:the\:bold\:numbers}[/tex]

[tex]\frac{\begin{matrix}\space\space&8&0&\textbf{2}\\ \times \:&\space\space&9&\textbf{4}\end{matrix}}{\begin{matrix}\space\space&\space\space&\space\space&8\end{matrix}}[/tex]

[tex]\frac{\begin{matrix}\space\space&8&\textbf{0}&2\\ \times \:&\space\space&9&\textbf{4}\end{matrix}}{\begin{matrix}\space\space&\space\space&0&8\end{matrix}}[/tex]

[tex]\mathrm{Multiply}\\\\8\cdot \:4=32[/tex]

[tex]\mathrm{Carry\:}3\mathrm{\:to\:the\:column\:on\:the\:left\:and\:write\:}2\mathrm{\:in\:the\:result\:line}[/tex]

[tex]\frac{\begin{matrix}\space\space&3&\space\space&\space\space&\space\space\\ \space\space&\space\space&\textbf{8}&0&2\\ \times \:&\space\space&\space\space&9&\textbf{4}\end{matrix}}{\begin{matrix}\space\space&\space\space&2&0&8\end{matrix}}[/tex]

[tex]\mathrm{Add\:the\:carried\:digit,\:}3\mathrm{,\:to\:the\:result}[/tex]

[tex]\frac{\begin{matrix}\space\space&3&\space\space&\space\space&\space\space\\ \space\space&\space\space&8&0&2\\ \times \:&\space\space&\space\space&9&4\end{matrix}}{\begin{matrix}\space\space&3&2&0&8\end{matrix}}[/tex]

[tex]\mathrm{Multiply\:the\:top\:number\:by\:the\:bolded\:digit\:of\:the\:bottom\:number}[/tex]

[tex]\frac{\begin{matrix}\space\space&\space\space&\textbf{8}&\textbf{0}&\textbf{2}\\ \space\space&\times \:&\space\space&\textbf{9}&4\end{matrix}}{\begin{matrix}\space\space&3&2&0&8\end{matrix}}[/tex]

[tex]\mathrm{Mutliply\:the\:bold\:numbers}:\quad \:2\cdot \:9=18[/tex]

[tex]\mathrm{Carry\:}1\mathrm{\:to\:the\:column\:on\:the\:left\:and\:write\:}8\mathrm{\:in\:the\:result\:line}[/tex]

[tex]\frac{\begin{matrix}\space\space&\space\space&\space\space&1&\space\space\\ \space\space&\space\space&8&0&\textbf{2}\\ \space\space&\times \:&\space\space&\textbf{9}&4\end{matrix}}{\begin{matrix}\space\space&3&2&0&8\\ \space\space&\space\space&\space\space&8&\space\space\end{matrix}}[/tex]

[tex]\mathrm{Add\:the\:carried\:number\:to\:the\:multiplication}:\quad \:1+0\cdot \:9=1[/tex]

[tex]\frac{\begin{matrix}\space\space&\space\space&\space\space&\textbf{1}&\space\space\\ \space\space&\space\space&8&\textbf{0}&2\\ \space\space&\times \:&\space\space&\textbf{9}&4\end{matrix}}{\begin{matrix}\space\space&3&2&0&8\\ \space\space&\space\space&1&8&\space\space\end{matrix}}[/tex]

[tex]\mathrm{Mutliply\:the\:bold\:numbers}:\quad \:8\cdot \:9=72[/tex]

[tex]\mathrm{Carry\:}7\mathrm{\:to\:the\:column\:on\:the\:left\:and\:write\:}2\mathrm{\:in\:the\:result\:line}[/tex]

[tex]\frac{\begin{matrix}\space\space&7&\space\space&1&\space\space\\ \space\space&\space\space&\textbf{8}&0&2\\ \times \:&\space\space&\space\space&\textbf{9}&4\end{matrix}}{\begin{matrix}\space\space&3&2&0&8\\ \space\space&2&1&8&\space\space\end{matrix}}[/tex]

[tex]\mathrm{Add\:the\:carried\:digit,\:}7\mathrm{,\:to\:the\:result}[/tex]

[tex]\frac{\begin{matrix}\space\space&7&\space\space&1&\space\space\\ \space\space&\space\space&8&0&2\\ \times \:&\space\space&\space\space&9&4\end{matrix}}{\begin{matrix}\space\space&3&2&0&8\\ 7&2&1&8&\space\space\end{matrix}}[/tex]

[tex]\mathrm{Add\:the\:rows\:to\:get\:the\:answer.\:\:\:Fill\:in\:trailing\:zeros\:\:(not\:required)}[/tex]

[tex]\frac{\begin{matrix}\space\space&\space\space&8&0&2\\ \space\space&\times \:&\space\space&9&4\end{matrix}}{\begin{matrix}0&3&2&0&8\\ 7&2&1&8&0\end{matrix}}[/tex]     [tex]=75,388[/tex]

See Image for the addition:

6^3 multiplied by (1/2)^3 ​

Answers

Step-by-step explanation:

6³×(½)³

=> 216÷8 = 27.

hope this helps you.

HELP ME PLEASE!!!!!!!!!!!

Answers

Answer:

SAS theorem >This is the correct

Answer:

SSS theorem

Step-by-step explanation:

[tex] \huge\colorbox{pink}{hope \: its \: help}[/tex]

The equilateral triangle with side 12 has altitude

Answers

Step by step explanation

There are 6 yellow, 9 blue, and 4 orange balls inside an urn. 8 balls are chosen from the urn without replacement. Let X, Y , and W denote the number of yellow, blue, and orange balls selected, respectively.

(a) Find P{X + W = 1}.
(b) Find P{X = 2, Y = W}

Answers

Using the probability concept, it is found that:

a) P{X + W = 1} = 0

b) P{X = 2, Y = W} = 0.0667

A probability is the number of desired outcomes divided by the number of total outcomes.

In this problem, the order in which the balls are chosen is not important, hence, the combination formula is used to find the number of outcomes.

Combination formula:

[tex]C_{n,x}[/tex] is the number of different combinations of x objects from a set of n elements, given by:

[tex]C_{n,x} = \frac{n!}{x!(n-x)!}[/tex]

In total, 8 balls are chosen from a set of 19, hence:

[tex]T = C_{19,8} = \frac{19!}{8!11!} = 75582[/tex]

Item a:

1 from a set of 15.7 from a set of 4.

Not possible, hence:

P{X + W = 1} = 0

Item b:

2 from a set of 6.3 from a set of 9.3 from a set of 4.

Hence:

[tex]D = C_{6,2}C_{9,3}C_{4,3} = \frac{6!}{2!4!} \times \frac{9!}{3!6!} \times \frac{4!}{3!1!} = 5040[/tex]

Then, the probability is:

[tex]p = \frac{D}{T} = \frac{5040}{75582} = 0.0667[/tex]

P{X = 2, Y = W} = 0.0667

A similar problem is given at https://brainly.com/question/15536019

which of the following are perfect cubes 1)3375 2)8000 3)6859​

Answers

Answer:

all are perfect cubes

Step-by-step explanation:

you can just cube root the numbers and bom if it aint decimal its a perfect cube

what is a degree 3 polynomial with integer coefficients with zeros 3, -4, 5

Answers

4 would it or maybe 12

2.
Fit a quadratic function to these three points:

(−2, 8), (0, −4), and (4, 68)

A. y = −2x2 − 2x − 4

B. y = 4x2 + 2x − 4

C. y = −4x2 − 2x − 4

D. y = −2x2 + 2x − 4

Answers

Answer:

It's C.

Step-by-step explanation:

Answer:

B

Step-by-step explanation:

just put the x and y values into the equations and see which equation stays true.

for example, point (4, 68) rules out A and C even without detailed calculation, because these 2 expressions can produce for positive x only negative y (and 68 is clearly a positive y).

so, when using this point in B :

68 = 4×4² + 2×4 - 4 = 4×16 + 8 - 4 = 64 + 8 - 4 = 68

correct.

and it is correct also for the other points.

so, B is the right answer.

Please tell me how much can 6 go into 14 without going over 14

Answers

2 times without leaving a remainder.

i need help on this math problem

Answers

Answer:

Step-by-step explanation:

Slope: 3/1 (positive: line go uphill)

y-intercept: -5

Place your pencil on the vertical line (y) on -5. Go up 3 units and 1 to the right, repeat and trace the line (actually you only need two points to trace a line)

7(x+7)≤56 plz help im bad at math

Answers

Answer:x[tex]\geq[/tex]1

Step-by-step explanation:

remove the brackets

7x+49≤56

collect like terms

7x≤56-49

7x[tex]\geq[/tex]7

divide both sides by 7

x[tex]\geq[/tex]1

100 Points
Emily wants to buy turquoise stones on her trip to New Mexico to give to at least 4 of her friends. The gift shop sells small stones for $4 and large stones for $6, and Emily has no more than $30 to spend.
Write and graph a system of Linear Equations and write two possible solutions.

Answers

Answer:

Step-by-step explanation:

what is she going to buy? Small stones or big stones? if its small stones then she can buy 7 small stones. If its big stones then she can buy 5 big stones.

PLS HELP REWARDING A LOT!!

Answers

Answer:

157°

Step-by-step explanation:

Sum of interior angles of polygon

(n-2)*180 ---- n = total number of sides

7 angles, 7 sides

5*180 = 900

final angle will be

900-162-115-125-138-105-98

=157

Answer:

157

Step-by-step explanation:

it is obvious that it's a heptagon

so (n-2) x180

n=7

(7-2) x 180 =900

900-162-115-125-138-105-98

900-743=157

final unknown=157°

Which expression is the factored form of−4.5n+3?


−1.5(3n+2)

−3(1.5n+1)

−3(1.5n−1)

−1.5(−3n+2)

Answers

Answer:

— 3 (1.5n — 1)

Step-by-step explanation:

[tex] - 4.5n + 3 = - \frac{9}{2} n + 3 = 3( - \frac{3}{2} n + 1) = - 3( \frac{3}{2}n - 1) \\ = - 3(1.5n - 1)[/tex]

one added to the square root of the sum of a number and one is equal to four. find the number​

Answers

[tex]\stackrel{\textit{one added to the }\sqrt{\qquad }}{1+\sqrt{\quad }}~~,~~ \stackrel{\sqrt{~~}\textit{ of a number and one}}{\sqrt{x+1}}\implies 1+\sqrt{x+1}\stackrel{\textit{equals}}{=}4 \\\\\\ \sqrt{x+1}=3\implies \left( \sqrt{x+1} \right)^2=3^2\implies x+1=9\implies \boxed{x=8}[/tex]

Which of the following could be a helpful first step in solving this equation for q?

p2q+2q=11
Question 5 options:

Combine p2q+2q

Divide the whole equation by 11.


Divide the first term by p2

Factor a q out of p2q+2q

Answers

Answer:

D: Factor a q out of p^2 q+ 2q

Step-by-step explanation:

I took the test. Hope it was helpful! :)

The first step in solving this equation p²q+2q=11 for q will be to factor q out of p²q+2q. Option D is correct.

What is the equation?

An equation is a statement that two expressions, which include variables and/or numbers, are equal. In essence, equations are questions, and efforts to systematically find solutions to these questions have been the driving forces behind the creation of mathematics.

It is given that, the equation is,

p²q+2q=11

Take q common from the L.H.S as

q(p²+2)=11

The value of q is obtained by rearranging the equation as,

q=11/(p²+2)

The value of q obtained after solving the equation p²q+2q=11 will be q=11/(p²+2).

Thus, the first step in solving this equation p²q+2q=11 for q will be to factor q out of p²q+2q. Option D is correct.

Learn more about the equation here,

https://brainly.com/question/10413253

#SPJ2

2 (3) to the power of 3 + 5

Answers

Answer:

13122

Step-by-step explanation:

Our equation is 2(3)⁸

once its solve we get 13122 as our answer

which option is right??

Answers

Side- Angle- Side
is the correct answer

hope this helps :)

Answer:

its the third choice

Step-by-step explanation:

Side angle side cuz there r 2 sides congruent in the picture shown above.

50 points + brainliest answer questions 7 and 8 with work and all

Answers

Answer:

7) m∠BHE = 146°

8) m∠BAC = 25°

Step-by-step explanation:

Question 7:

Given that, [tex]\displaystyle\mathsf{\overline{CD}\:||\:\overline{EF}}[/tex], and that [tex]\displaystyle\mathsf{\overline{AB}}[/tex] is a transveral.

We are also provided with the following measures of the angles: m∠DGH = 2x, and m∠FHB = 5x - 51.

∠DGH and ∠FHB are also corresponding angles, as they have corresponding positions on the same side of the transversal, [tex]\displaystyle\mathsf{\overline{AB}}[/tex]. These two angles also have the same measure.  

Solve for x:

In order to find the measure of ∠BHE, we could set up an equality statement on ∠DGH and ∠FHB to solve for the value of x.

m∠DGH = m∠FHB

2x = 5x - 51

Add 51 and subtract 2x from both sides of the equation:

2x -2x + 51  = 5x - 2x - 51 + 51

51 = 3x

Divide both sides by 3:

[tex]\displaystyle\mathsf{\frac{51}{3}\:=\:\frac{3x}{3}}[/tex]

x = 17

 m∠DGH = 2x = 2(17) = 34°,

 m∠FHB = 5x - 51 = 5(17) - 51 = 34°.

Since ∠FHB and ∠BHE are supplements (whose sum add up to 180°), we could determine the measure of ∠BHE as follows:

m∠BHE + m∠FHB  = 180°

m∠BHE + 34° = 180°

m∠BHE + 34°- 34° = 180°- 34°

m∠BHE = 146°.

Therefore, the measure of ∠BHE is 146°.

Question 8:

Given that ΔABC with [tex]\displaystyle\mathsf{\overline{AC}}[/tex] extended to D, and that m∠ABC = 63° and m∠BCD = 92°:

The Exterior Angle Theorem states that the measure of an exterior angle of a triangle is equal to the sum of the two nonadjacent interior angles. In other words: ∠BCD = ∠BAC + ∠ABC.  

Before we could apply the Exterior Angle Theorem, we must first find m∠BAC. Since ∠BCD and ∠BCA are supplements:

m∠BCD + m∠BCA = 180°

92° + m∠BCA = 180°

92° - 92°+ m∠BCA = 180° - 92°

m∠BCA = 88°

Solve for m∠BAC:

Now that we have the measure for ∠BCA, we can find m∠BAC by applying the Triangle Sum Theorem where it states that the sum of the interior angles of a triangle is equal to 180°.

m∠BCD + m∠ABC + m∠BAC = 180°

92° + 63° + m∠BAC  = 180°

125° + m∠BAC  = 180°

155° - 155° + m∠BAC  = 180° - 155°

m∠BAC  = 25°

Therefore, the measure of ∠BAC is 25°.

Y=4x+8 y=-x-7 solve by substitution

Answers

Step-by-step explanation:

Alrighty! here you go image.

Answer:x=15

Step-by-step explanation:

Flying against the wind, a jet travels 3780 miles in 6 hours. Flying with the wind, the same jet travels 5950miles in 7 hours. What is the rate of the jet in still air and what is the rate of the wind? 1111 Rate of the jet in still air: ​

Answers

The rate of the jet in still air is 740 mph and the rate of the wind is 110 mph.

The given parameters;

distance traveled against the wind, d₁ = 3780 milestime of motion, t₁ = 6 hoursdistance traveled with wind, d₂ = 5950 milestime of motion, t₂ = 7 hours

Let the rate of the jet in still air = R₁

Let the rate of the wind = R₂

The rate of the jet against the wind:

[tex]R_1 - R_2 = \frac{3780}{6} \\\\R_1 - R_2 = 630[/tex]

The rate of the jet with the wind:

[tex]R_1 + R_2 = \frac{5950}{7} \\\\R_1 + R_2 = 850\\\\[/tex]

The rate of the jet in still air is calculated as follows;

[tex]R_1 = \frac{630 + 850}{2} \\\\R_1 = 740 \ mph[/tex]

The rate of the wind is calculated as follows;

[tex]R_1 - R_2 = 630\\\\R_2 = R_1 - 630\\\\R_2 = 740 - 630 \\\\R_2 = 110 \ mph[/tex]

Thus, the rate of the jet in still air is 740 mph and the rate of the wind is 110 mph.

Learn more about rate here:https://brainly.com/question/11664428

2 - 2x 4x + 12
------- = ---------
4 3

Answers

[tex]\tt\frac { 2 - 2 x } { 4 } = \frac { 4 x + 12 } { 3 }[/tex]

[tex]\tt3\left(2-2x\right)=4\left(4x+12\right) [/tex]

[tex]\tt6-6x=16x+48 [/tex]

[tex]\tt6-6x-16x=48 [/tex]

[tex]\tt6-22x=48 [/tex]

[tex]\tt-22x=42 [/tex]

[tex]\tt\:x=\frac{42}{-22} [/tex]

[tex]\boxed{\tt\:x = -\frac{21}{11} = -1\frac{10}{11} \approx -1.90}[/tex]

Method used :-

◇ ✖ Cross multiplication ❌

________

Hope it helps ⚜

Step-by-step explanation:

Given equation is [{(2-2x)/4} = {(4x+12)/3]

On applying cross multiplication then

Since, (a/b) = (c/d)

⇛ab = bc

Where, a = (2-2x), b = (4), c = (4x+12) and d = (3).

Now,

⇛3(2-2x) = 4(4x+12)

Multiply the number outside the bracket with numbers and variables in the bracket.

⇛6 - 6x = 16x + 48

Shift the variable value on LHS and constant on RHS.

⇛-6x - 16x = 48-6

Subtract the values on LHS and RHS.

⇛-22x = 42

Shift the value (-22) from LHS to RHS.

⇛x = 42/-22

Write the obtained answer in lowest form by cancelling method.

⇛x = (42÷2)/(-22÷2)

Therefore, x = 21/-11 = -(21/11)

Answer: Hence the value of x for the given problem is -21/11.

VERIFICATION:

If x = -21/11 then the equation is

[{(2-2x)/4} = {(4x+12)/3]

Substitute the value x = -21/11 in expression, then

⇛[{2-2(-21/11)}/4 = {4(-21/11)+12}/3]

Multiply (-2) from (-21) to get 42 on LHS.

⇛[{2(42/11)}/4 = {4(-21/11)+12}/3]

Again multiply 4 from (-21) to get 81 on RHS.

⇛[{2(42/11)}/4 = {(-81/11) + 12}/3]

Take the LCM of the denominator 1 and 11 is 11 on LHS

⇛[{(2*11 + 42*1)/11}/4 = {(-81*1 + 12*11)/11}/3]

Multiply the numerator on both LHS and RHS.

⇛[{(22+42)/11}/4 = {(-81 + 132)/11}/3]

Add the numerator numbers on both LHS and RHS

⇛[{(64/11)}/4 = [{(48/11)}/3]

Now, conver the division fraction in multiply fraction on both LHS and RHS.

⇛[{(64/11) * 4} = {(48/11) * 3}]

Now, write the numbers in Lower form by cancelling method, on both LHS and RHS.

⇛(16/11) = (16/11)

LHS = RHS

Please let me know if you have any other questions.

-7(8) What is it please help me I need help help help help

Answers

The Answer should be 10

List all the factors pairs in the table 64

Answers

(1, 64), (2, 32), (4, 16), (8, 8).

Answer:

1 x 64

2 x 32

4 x 16

8 x 8

Step-by-step explanation:

Factors of 64: 1, 2, 4, 8, 16, 32 and 64.

Pairs:

1 x 64

2 x 32

4 x 16

8 x 8

Hope this helps :)

Other Questions
Fill in the correct answers solve pls brainliest Drag each item to indicate whether it exhibits fracture or cleavage. What is important team leaders training what are some ways to get a cat to stop meowing? my moms cat has been meowing all morning and its giving me a headache. i dont know if he is upset, hungry or just wants something but he cant be hungry cause he just ate. The Constitution ensures that political power is split among the branches of government. The ability of Congress to impeach and remove the president is a check on the power of the president. How do you think this check on the president strengthens democracy? A quadratic function has zeros of 2 and 9. Which expression could define the function?A. x+18x+11B. x+11x+18C. x-11x+18D. x-18x+11- Also could you show the work. I am trying to do test corrections and have no idea what I am doing. If the population of a country is 1,000,000 people, its labor force consists of 500,000, and 40,000 people are unemployed, the unemployment rate is: who the last king of Aksum What is the sum of the first 10 terms of the geometric series 2 + 6 + 18 + 54 + 162 + ...? When thirteen is reduced by two-thirds of a number, the result is 7. Find the number.The number is QUESTION 7All of the following cells might be a product of meiosis EXCEPT?OA) an eggO B) a liver cellOC) a sporeOD) a sperm cellE) a gamete Friction is similar to drag. Like drag it acts on an object in the opposite direction from the objects motion. How does the friction slow motion? mention any five importance of child rights in the presence of a regulatory protein the lac operon is _____. the role of haemoglobin in the transport of oxygen and carbon dioxide Lyn, Sarah, and Keanna Which statements best describes how William kamkwama feels at the beginning of his memoir? Audrey and her sister found 9 dimes and 8 pennies. If they share the money equally, how muchmoney will each sister get? Find two positive numbers whose product is 300 and such that the sum of the first and four times the second is a minimum. Why does an explorer have to have some patience?